4
$\begingroup$

Let $A\in \mathbb{R}^{n\times n}$ be a positive definite symmetric matrix with eigenvalues $\lambda_1\ge \cdots\ge \lambda_n$, $X\in \mathbb{R}^{n\times k}$ such that $X'X=I_k$ ($X'$ means the transpose of $X$) and $n\ge 2k$, then $$\det(X'AXX'A^{-1}X)\le \prod\limits_{j=1}^k\frac{(\lambda_j+\lambda_{n-j+1})^2}{4\lambda_j\lambda_{n-j+1}}.$$

This result was first proved by Bloomfield and Watson(1975) and Knott(1975). I came across a note by H. Yang (A brief proof on the generalized variance bound of the relative efficiency in statistics, Communications in Statistics - Theory and Methods, 19(1990):12, 4587-4590), but the notation of his proof was rather confusing. Can any one make a clean proof or explain his proof?

(I don't have an e-version of that paper, sorry for inconvenience)

$\endgroup$

1 Answer 1

3
$\begingroup$

Here is a crude sketch that could work. First recall Hadamard's determinant theorem:

\begin{eqnarray*} |X'AX| \le \prod_{i}(X'AX)_{ii} = \prod_i x_i^TAx_i. \end{eqnarray*}

Now recall Kantorovich's inequality (for $x^Tx=1$ ) \begin{eqnarray*} (x^TAx)(x^TA^{-1}x) \le \frac{(\lambda_1 + \lambda_n)^2}{4\lambda_1\lambda_n} \end{eqnarray*}

Now, inductively apply this inequality to the product

\begin{eqnarray*} \prod_i x_i^TAx_i \end{eqnarray*} by using $x_i^Tx_j = 0$, if $i\neq j$, so that for example, we have \begin{eqnarray*} (x_1^TAx_1)(x_1^TA^{-1}x_1) &\le& \frac{(\lambda_1 + \lambda_n)^2}{4\lambda_1\lambda_n}\\\\ (x_2^TAx_2)(x_2^TA^{-1}x_2) &\le& \frac{(\lambda_2 + \lambda_{n-1})^2}{4\lambda_2\lambda_{n-1}}, \end{eqnarray*} and so on uptil $x_k$.

$\endgroup$
3
  • $\begingroup$ It is not clear how you get \begin{eqnarray*} (x_2^TAx_2)(x_2^TA^{-1}x_2) &\le& \frac{(\lambda_2 + \lambda_{n-1})^2}{4\lambda_2\lambda_{n-1}}. \end{eqnarray*} $\endgroup$
    – Sunni
    Sep 2, 2011 at 0:18
  • $\begingroup$ Btw, did you read that article by Yang 1990? $\endgroup$
    – Sunni
    Sep 2, 2011 at 0:19
  • $\begingroup$ I have not yet worked out the details, but "it could work"; as for Yang, nope, I have not read any of the papers that you have cited; no time. $\endgroup$
    – Suvrit
    Sep 2, 2011 at 0:46

Your Answer

By clicking “Post Your Answer”, you agree to our terms of service and acknowledge you have read our privacy policy.

Not the answer you're looking for? Browse other questions tagged or ask your own question.